Last visit was: 05 May 2024, 01:57 It is currently 05 May 2024, 01:57

Close
GMAT Club Daily Prep
Thank you for using the timer - this advanced tool can estimate your performance and suggest more practice questions. We have subscribed you to Daily Prep Questions via email.

Customized
for You

we will pick new questions that match your level based on your Timer History

Track
Your Progress

every week, we’ll send you an estimated GMAT score based on your performance

Practice
Pays

we will pick new questions that match your level based on your Timer History
Not interested in getting valuable practice questions and articles delivered to your email? No problem, unsubscribe here.
Close
Request Expert Reply
Confirm Cancel
SORT BY:
Date
Tags:
Show Tags
Hide Tags
User avatar
Manager
Manager
Joined: 07 Jan 2010
Posts: 80
Own Kudos [?]: 297 [7]
Given Kudos: 57
Location: So. CA
Concentration: General
WE 1: 2 IT
WE 2: 4 Software Analyst
Send PM
Intern
Intern
Joined: 01 Dec 2010
Posts: 3
Own Kudos [?]: 2 [2]
Given Kudos: 9
Send PM
User avatar
Director
Director
Joined: 08 May 2009
Status:There is always something new !!
Affiliations: PMI,QAI Global,eXampleCG
Posts: 550
Own Kudos [?]: 589 [1]
Given Kudos: 10
Send PM
User avatar
Intern
Intern
Joined: 27 Jun 2008
Posts: 33
Own Kudos [?]: 28 [0]
Given Kudos: 22
Location: United States (AL)
Concentration: General Management, Technology
GMAT 1: 660 Q48 V34
WE:Consulting (Computer Software)
Send PM
Re: Measurements of the motion of the planet Uranus seem to show [#permalink]
B it is..

it gives an alternate explanation than what is stated in the question stem.

Originally posted by kanhaiya on 02 Dec 2010, 01:24.
Last edited by kanhaiya on 02 Dec 2010, 01:33, edited 1 time in total.
User avatar
Manager
Manager
Joined: 07 Jan 2010
Posts: 80
Own Kudos [?]: 297 [0]
Given Kudos: 57
Location: So. CA
Concentration: General
WE 1: 2 IT
WE 2: 4 Software Analyst
Send PM
Re: Measurements of the motion of the planet Uranus seem to show [#permalink]
ah, ok your explanations help a lot. i wasn't connecting the conclusion to the g-pull, the alternate explanation makes sense now. the whole time i was thinking that the answer had to do with the g-pull which i narrowed down to choice (b) and (d)
User avatar
Manager
Manager
Joined: 02 Nov 2010
Posts: 71
Own Kudos [?]: 56 [0]
Given Kudos: 15
Location: india
Concentration: Finance,general management
 Q49  V27 GMAT 2: 680  Q47  V35
WE 1: 1.3
Send PM
Re: Measurements of the motion of the planet Uranus seem to show [#permalink]
if b is correct then question must that 'comets exist beyond uranus' and not
' beyond the orbit of Pluto'
p n u sun
Tutor
Joined: 16 Oct 2010
Posts: 14871
Own Kudos [?]: 65104 [1]
Given Kudos: 431
Location: Pune, India
Send PM
Re: Measurements of the motion of the planet Uranus seem to show [#permalink]
1
Bookmarks
Expert Reply
adhithya wrote:
if b is correct then question must that 'comets exist beyond uranus' and not
' beyond the orbit of Pluto'
p n u sun


The argument tells us that Neptune and Pluto are beyond Uranus. Since they are not exerting the force on Uranus, the conclusion is that there is another planet beyond Pluto that we haven't yet discovered.
Option (B) tells that they there is a comet belt beyond Pluto with powerful pull. Hence, instead of a planet, the comets might be pulling Uranus away.
User avatar
Manager
Manager
Joined: 23 Sep 2009
Posts: 71
Own Kudos [?]: 530 [0]
Given Kudos: 37
Send PM
Re: Measurements of the motion of the planet Uranus seem to show [#permalink]
Answer B tells that it is not Neptune or Pluto, but some other comets that exert the pull on Uranus...
In weaking questions we can have a choice which gives another reason.
So (B)
avatar
Manager
Manager
Joined: 13 Jul 2010
Posts: 81
Own Kudos [?]: 218 [0]
Given Kudos: 7
Send PM
Re: Measurements of the motion of the planet Uranus seem to show [#permalink]
Answer B provides the weakener we are looking for. Something else besides another planet is pulling the outer planets (Uranus, Neptune, and Pluto). Notice the stem only mentions the inner planets so all 3 of these outer planets are subject to this pull.
avatar
Intern
Intern
Joined: 12 Aug 2010
Posts: 41
Own Kudos [?]: 88 [0]
Given Kudos: 50
Concentration: Finance, Marketing
Schools:UNC Kenan-Flagler, IU Kelley, Emory GSB
WE 1: 5 yrs
Send PM
Re: Measurements of the motion of the planet Uranus seem to show [#permalink]
VeritasPrepKarishma wrote:
adhithya wrote:
if b is correct then question must that 'comets exist beyond uranus' and not
' beyond the orbit of Pluto'
p n u sun


The argument tells us that Neptune and Pluto are beyond Uranus. Since they are not exerting the force on Uranus, the conclusion is that there is another planet beyond Pluto that we haven't yet discovered.
Option (B) tells that they there is a comet belt beyond Pluto with powerful pull. Hence, instead of a planet, the comets might be pulling Uranus away.


Hi Karishma,

But why isn't D good? D states that sun itself exerts lesser pull on uranus - so the cause is not the presence of a planet, its the absence of sun's pull itself...this very much undermines the argument...

eg. A jaguar is a very expensive car. Since John bought a jaguar, he mustv'e been helped financially by X.
Here the assumption is that everyone pays same for Jaguar. But if it so happens that the company charges very little from John, the argument stands weakened.
Though a statement such as: 'Z helps John too' also undermines the argument, I found the former better.

I really cant pinpoint the difference between B and D.
Tutor
Joined: 16 Oct 2010
Posts: 14871
Own Kudos [?]: 65104 [2]
Given Kudos: 431
Location: Pune, India
Send PM
Re: Measurements of the motion of the planet Uranus seem to show [#permalink]
2
Kudos
Expert Reply
yossarian84 wrote:

Hi Karishma,

But why isn't D good? D states that sun itself exerts lesser pull on uranus - so the cause is not the presence of a planet, its the absence of sun's pull itself...this very much undermines the argument...

eg. A jaguar is a very expensive car. Since John bought a jaguar, he mustv'e been helped financially by X.
Here the assumption is that everyone pays same for Jaguar. But if it so happens that the company charges very little from John, the argument stands weakened.
Though a statement such as: 'Z helps John too' also undermines the argument, I found the former better.

I really cant pinpoint the difference between B and D.



The issue at hand is "Measurements of the motion of the planet Uranus seem to show Uranus being tugged by a force pulling it away from the Sun and the inner planets." - the first line of the question. Uranus is being tugged by a force opposite to the direction of the Sun's pull. We are interested in the source of that pull. It doesn't matter how strong the Sun's pull is. If Sun's pull is weaker, the opposite force doesn't need to be very strong. If Sun's pull is stronger, the opposite force is even stronger. Bottom line, we need to find the source of the pull which is acting opposite to the Sun's pull. The argument says that the source of that pull cannot be the outer planets since their mass isn't enough. Hence there must be another planet beyond Pluto. But we undermine the argument by saying that there are comets with powerful pull beyond Pluto. So they could be the source rather than a planet yet to be discovered.
User avatar
Verbal Forum Moderator
Joined: 31 Jan 2010
Posts: 312
Own Kudos [?]: 343 [0]
Given Kudos: 149
 Q49  V42
WE 1: 4 years Tech
Send PM
Re: Measurements of the motion of the planet Uranus seem to show [#permalink]
billnepill wrote:
gtr022001 wrote:
Measurements of the motion of the planet Uranus seem to show Uranus being tugged by a force pulling it away from the Sun and the inner planets. Neptune and Pluto, the two known planets whose orbits are farther from the Sun than is the orbit of Uranus, do not have enough mass to exert the force that the measurements indicate. Therefore, in addition to the known planets, there must be at least one planet in our solar system that we have yet to discover.

Which one of the following, if true, most seriously weakens the argument?

(A) Pluto was not discovered until 1930
(B) There is a belt of comets beyond the orbit of Pluto with powerful gravitational pull.
(C) Neither Neptune nor Pluto is as massive as Uranus.
(D) The force the Sun exerts on Uranus is weaker than the force it exerts on the inner planets.
(E) Uranus' orbit is closer to Neptune's orbit than it is to Pluto's.

i narrowed it down to 2 choices in which the rest are out of scope, but i really don't understand the argument. maybe i got lucky with the answer.


This is a weaken question. So, we have to find the conclusion and attack it.

Premise: Uranus pulled away from the sun and inner planets
Premise: Neptune and Pluto are farther from the sun, but no mass to attract Uranus
Conclusion :Therefore, there must be an undiscovered planet pulling Uranus away by gravitation forces

D) The force the Sun exerts on Uranus is weaker than the force it exerts on the inner planets. No effect on the conclusion. We already know that some other than the sun object influences Uranus movement. We just have to make sure it's not a new planet

It is nowhere mentioned in the stimulus that some other than the sun object influences uranus movement .The stimulus says "seem to show Uranus being tugged by a force pulling it away from the Sun and the inner planets"
So what if no force that pulls uranus away actually exists , It is the sun that pulls all the all the planets toward itself . Uranus is the only planet for which the suns force is not strong enough
Tutor
Joined: 16 Oct 2010
Posts: 14871
Own Kudos [?]: 65104 [0]
Given Kudos: 431
Location: Pune, India
Send PM
Re: Measurements of the motion of the planet Uranus seem to show [#permalink]
Expert Reply
mundasingh123 wrote:
It is nowhere mentioned in the stimulus that some other than the sun object influences uranus movement .The stimulus says "seem to show Uranus being tugged by a force pulling it away from the Sun and the inner planets"
So what if no force that pulls uranus away actually exists , It is the sun that pulls all the all the planets toward itself . Uranus is the only planet for which the suns force is not strong enough


Argument:
Measurements of the motion of the planet Uranus seem to show Uranus being tugged by a force pulling it away from the Sun and the inner planets. Neptune and Pluto, the two known planets whose orbits are farther from the Sun than is the orbit of Uranus, do not have enough mass to exert the force that the measurements indicate. Therefore, in addition to the known planets, there must be at least one planet in our solar system that we have yet to discover.

Look at the colored portion above.. Measurements indicate that a force is acting to pull Uranus away from the Sun... (When no force acts on a body, it maintains straight line motion (or something like that, I dont remember my Physics well).. Sun acts on Uranus because of which Uranus revolves around the sun. How strong this force is doesn't matter because it is enough to make Uranus revolve around the Sun. If this force is less than that which Sun exerts on the inner planets, it doesn't bother us (as suggested in option D). The argument doesn't say that strength of the force that Sun exerts has changed or is decreasing.) Measurements are indicating another force acting against the Sun's force. So we are looking for the source of this force acting against the Sun's force.
User avatar
Verbal Forum Moderator
Joined: 31 Jan 2010
Posts: 312
Own Kudos [?]: 343 [0]
Given Kudos: 149
 Q49  V42
WE 1: 4 years Tech
Send PM
Re: Measurements of the motion of the planet Uranus seem to show [#permalink]
VeritasPrepKarishma wrote:
mundasingh123 wrote:
It is nowhere mentioned in the stimulus that some other than the sun object influences uranus movement .The stimulus says "seem to show Uranus being tugged by a force pulling it away from the Sun and the inner planets"
So what if no force that pulls uranus away actually exists , It is the sun that pulls all the all the planets toward itself . Uranus is the only planet for which the suns force is not strong enough


Argument:
Measurements of the motion of the planet Uranus seem to show Uranus being tugged by a force pulling it away from the Sun and the inner planets. Neptune and Pluto, the two known planets whose orbits are farther from the Sun than is the orbit of Uranus, do not have enough mass to exert the force that the measurements indicate. Therefore, in addition to the known planets, there must be at least one planet in our solar system that we have yet to discover.

Look at the colored portion above.. Measurements indicate that a force is acting to pull Uranus away from the Sun... (When no force acts on a body, it maintains straight line motion (or something like that, I dont remember my Physics well).. Sun acts on Uranus because of which Uranus revolves around the sun. How strong this force is doesn't matter because it is enough to make Uranus revolve around the Sun. If this force is less than that which Sun exerts on the inner planets, it doesn't bother us (as suggested in option D). The argument doesn't say that strength of the force that Sun exerts has changed or is decreasing.) Measurements are indicating another force acting against the Sun's force. So we are looking for the source of this force acting against the Sun's force.

Hi Karishma thanks for taking the time out to answer the question raised by a number of people repeatedly.
Lastly , can you tell us how should we consider the words "seem to " in bold in the quoted para above.Should we interpret "seem to " as apparently or actually.
Tutor
Joined: 16 Oct 2010
Posts: 14871
Own Kudos [?]: 65104 [0]
Given Kudos: 431
Location: Pune, India
Send PM
Re: Measurements of the motion of the planet Uranus seem to show [#permalink]
Expert Reply
mundasingh123 wrote:
VeritasPrepKarishma wrote:
mundasingh123 wrote:
It is nowhere mentioned in the stimulus that some other than the sun object influences uranus movement .The stimulus says "seem to show Uranus being tugged by a force pulling it away from the Sun and the inner planets"
So what if no force that pulls uranus away actually exists , It is the sun that pulls all the all the planets toward itself . Uranus is the only planet for which the suns force is not strong enough


Argument:
Measurements of the motion of the planet Uranus seem to show Uranus being tugged by a force pulling it away from the Sun and the inner planets. Neptune and Pluto, the two known planets whose orbits are farther from the Sun than is the orbit of Uranus, do not have enough mass to exert the force that the measurements indicate. Therefore, in addition to the known planets, there must be at least one planet in our solar system that we have yet to discover.

Look at the colored portion above.. Measurements indicate that a force is acting to pull Uranus away from the Sun... (When no force acts on a body, it maintains straight line motion (or something like that, I dont remember my Physics well).. Sun acts on Uranus because of which Uranus revolves around the sun. How strong this force is doesn't matter because it is enough to make Uranus revolve around the Sun. If this force is less than that which Sun exerts on the inner planets, it doesn't bother us (as suggested in option D). The argument doesn't say that strength of the force that Sun exerts has changed or is decreasing.) Measurements are indicating another force acting against the Sun's force. So we are looking for the source of this force acting against the Sun's force.

Hi Karishma thanks for taking the time out to answer the question raised by a number of people repeatedly.
Lastly , can you tell us how should we consider the words "seem to " in bold in the quoted para above.Should we interpret "seem to " as apparently or actually.


'Measurements seem to show' means that 'measurements are indicating'... Whether there actually is a force pulling the planet or not is not established. It is a hypothesis based on the measurements.

e.g. Her behavior seems to show she is in trouble.
'indication received from her behavior'

If option D were: The force the Sun exerts on Uranus is decreasing every year or something, our conclusion might have been weakened. But D only compares the force exerted on Uranus with the force exerted on other planets which is irrelevant to us.
User avatar
Senior Manager
Senior Manager
Joined: 21 Dec 2010
Posts: 267
Own Kudos [?]: 1333 [0]
Given Kudos: 51
Send PM
Re: Measurements of the motion of the planet Uranus seem to show [#permalink]
this is a simple B , took 1:03 mins , may be because i remember my physics very well. sometimes familiarity with the subject matter really helps, at other times some really awkward subject comes up and takes away too much time
Current Student
Joined: 31 Jul 2017
Status:He came. He saw. He conquered. -- Going to Business School -- Corruptus in Extremis
Posts: 1734
Own Kudos [?]: 5755 [0]
Given Kudos: 3063
Location: United States (MA)
Concentration: Finance, Economics
Send PM
Re: Measurements of the motion of the planet Uranus seem to show [#permalink]
Expert Reply
OA added and bumping for further discussion
Senior Manager
Senior Manager
Joined: 31 Jan 2019
Posts: 368
Own Kudos [?]: 711 [0]
Given Kudos: 67
Location: Switzerland
Concentration: General Management
GPA: 3.9
Send PM
Re: Measurements of the motion of the planet Uranus seem to show [#permalink]
Measurements of the motion of the planet Uranus seem to show Uranus being tugged by a force pulling it away from the Sun and the inner planets. Neptune and Pluto, the two known planets whose orbits are farther from the Sun than is the orbit of Uranus, do not have enough mass to exert the force that the measurements indicate. Therefore, in addition to the known planets, there must be at least one planet in our solar system that we have yet to discover.

Which one of the following, if true, most seriously weakens the argument?


Weaken question

Pre-thinking

Let's try to work on the assumptions

Falsification scenario: Something other than a planet is responsible for the movements

Assumption: Nothing but planets external to Uranus can pull Uranus

A correct weakener will work with our falsification scenario

POE

(A) Pluto was not discovered until 1930
Irrelevant

(B) There is a belt of comets beyond the orbit of Pluto with powerful gravitational pull.
In line with ou falsification scenario

(C) Neither Neptune nor Pluto is as massive as Uranus.
Irrelevant

(D) The force the Sun exerts on Uranus is weaker than the force it exerts on the inner planets.
Irrelevant

(E) Uranus' orbit is closer to Neptune's orbit than it is to Pluto's.
Irrelevant

Intern
Intern
Joined: 09 Mar 2018
Posts: 17
Own Kudos [?]: 6 [0]
Given Kudos: 73
Send PM
Re: Measurements of the motion of the planet Uranus seem to show [#permalink]
We have to find an option which mentions something is pulling Uranus but not planet

Option B conveys aptly

Posted from my mobile device
CEO
CEO
Joined: 07 Mar 2019
Posts: 2563
Own Kudos [?]: 1822 [0]
Given Kudos: 763
Location: India
WE:Sales (Energy and Utilities)
Send PM
Re: Measurements of the motion of the planet Uranus seem to show [#permalink]
Measurements of the motion of the planet Uranus seem to show Uranus being tugged by a force pulling it away from the Sun and the inner planets. Neptune and Pluto, the two known planets whose orbits are farther from the Sun than is the orbit of Uranus, do not have enough mass to exert the force that the measurements indicate. Therefore, in addition to the known planets, there must be at least one planet in our solar system that we have yet to discover.

Which one of the following, if true, most seriously weakens the argument?

(A) Pluto was not discovered until 1930 - WRONG. Nothing done. Irrelevant.
(B) There is a belt of comets beyond the orbit of Pluto with powerful gravitational pull. - CORRECT. This may have lead to the pull opposite to Sun's.
(C) Neither Neptune nor Pluto is as massive as Uranus. - WRONG. Already said in passage.
(D) The force the Sun exerts on Uranus is weaker than the force it exerts on the inner planets. - WRONG. Scope shift and it's a trap.
(E) Uranus' orbit is closer to Neptune's orbit than it is to Pluto's. - WRONG. Again it does not impact the conclusion of the passage.

Answer B.
GMAT Club Bot
Re: Measurements of the motion of the planet Uranus seem to show [#permalink]
Moderators:
GMAT Club Verbal Expert
6925 posts
GMAT Club Verbal Expert
238 posts
CR Forum Moderator
832 posts

Powered by phpBB © phpBB Group | Emoji artwork provided by EmojiOne